Can someone help me solve this limit?












0












$begingroup$


Can someone help me solve this limit? Thank you in advance.



$$lim_{xto 0} bigg( frac{1+tan{x}}{1+sin{x}} bigg)^{frac{1}{sin{x}}}$$



If possible without L'Hospital, the exercise gives more points if we solve it without it.










share|cite|improve this question









New contributor




user644728 is a new contributor to this site. Take care in asking for clarification, commenting, and answering.
Check out our Code of Conduct.







$endgroup$








  • 1




    $begingroup$
    Looks like a case for the rule of L'Hospital. Have you tried that?
    $endgroup$
    – Cornman
    3 hours ago










  • $begingroup$
    Hint: $sin x$ and $tan x$ can be replaced by $x$, to the first order. The limit is that of $(1+o(x))^{1/x}$.
    $endgroup$
    – Yves Daoust
    3 hours ago












  • $begingroup$
    I have forgot to add the last part that if it is possible to not use L'Hospital , because if we use it in places where they think we could have used something else , they penalise us
    $endgroup$
    – user644728
    3 hours ago










  • $begingroup$
    "Can someone help me solve this limit?" And how is that supposed to help you learn?
    $endgroup$
    – John Coleman
    2 hours ago
















0












$begingroup$


Can someone help me solve this limit? Thank you in advance.



$$lim_{xto 0} bigg( frac{1+tan{x}}{1+sin{x}} bigg)^{frac{1}{sin{x}}}$$



If possible without L'Hospital, the exercise gives more points if we solve it without it.










share|cite|improve this question









New contributor




user644728 is a new contributor to this site. Take care in asking for clarification, commenting, and answering.
Check out our Code of Conduct.







$endgroup$








  • 1




    $begingroup$
    Looks like a case for the rule of L'Hospital. Have you tried that?
    $endgroup$
    – Cornman
    3 hours ago










  • $begingroup$
    Hint: $sin x$ and $tan x$ can be replaced by $x$, to the first order. The limit is that of $(1+o(x))^{1/x}$.
    $endgroup$
    – Yves Daoust
    3 hours ago












  • $begingroup$
    I have forgot to add the last part that if it is possible to not use L'Hospital , because if we use it in places where they think we could have used something else , they penalise us
    $endgroup$
    – user644728
    3 hours ago










  • $begingroup$
    "Can someone help me solve this limit?" And how is that supposed to help you learn?
    $endgroup$
    – John Coleman
    2 hours ago














0












0








0





$begingroup$


Can someone help me solve this limit? Thank you in advance.



$$lim_{xto 0} bigg( frac{1+tan{x}}{1+sin{x}} bigg)^{frac{1}{sin{x}}}$$



If possible without L'Hospital, the exercise gives more points if we solve it without it.










share|cite|improve this question









New contributor




user644728 is a new contributor to this site. Take care in asking for clarification, commenting, and answering.
Check out our Code of Conduct.







$endgroup$




Can someone help me solve this limit? Thank you in advance.



$$lim_{xto 0} bigg( frac{1+tan{x}}{1+sin{x}} bigg)^{frac{1}{sin{x}}}$$



If possible without L'Hospital, the exercise gives more points if we solve it without it.







real-analysis calculus limits






share|cite|improve this question









New contributor




user644728 is a new contributor to this site. Take care in asking for clarification, commenting, and answering.
Check out our Code of Conduct.











share|cite|improve this question









New contributor




user644728 is a new contributor to this site. Take care in asking for clarification, commenting, and answering.
Check out our Code of Conduct.









share|cite|improve this question




share|cite|improve this question








edited 3 hours ago









Lemniscate

402211




402211






New contributor




user644728 is a new contributor to this site. Take care in asking for clarification, commenting, and answering.
Check out our Code of Conduct.









asked 3 hours ago









user644728user644728

325




325




New contributor




user644728 is a new contributor to this site. Take care in asking for clarification, commenting, and answering.
Check out our Code of Conduct.





New contributor





user644728 is a new contributor to this site. Take care in asking for clarification, commenting, and answering.
Check out our Code of Conduct.






user644728 is a new contributor to this site. Take care in asking for clarification, commenting, and answering.
Check out our Code of Conduct.








  • 1




    $begingroup$
    Looks like a case for the rule of L'Hospital. Have you tried that?
    $endgroup$
    – Cornman
    3 hours ago










  • $begingroup$
    Hint: $sin x$ and $tan x$ can be replaced by $x$, to the first order. The limit is that of $(1+o(x))^{1/x}$.
    $endgroup$
    – Yves Daoust
    3 hours ago












  • $begingroup$
    I have forgot to add the last part that if it is possible to not use L'Hospital , because if we use it in places where they think we could have used something else , they penalise us
    $endgroup$
    – user644728
    3 hours ago










  • $begingroup$
    "Can someone help me solve this limit?" And how is that supposed to help you learn?
    $endgroup$
    – John Coleman
    2 hours ago














  • 1




    $begingroup$
    Looks like a case for the rule of L'Hospital. Have you tried that?
    $endgroup$
    – Cornman
    3 hours ago










  • $begingroup$
    Hint: $sin x$ and $tan x$ can be replaced by $x$, to the first order. The limit is that of $(1+o(x))^{1/x}$.
    $endgroup$
    – Yves Daoust
    3 hours ago












  • $begingroup$
    I have forgot to add the last part that if it is possible to not use L'Hospital , because if we use it in places where they think we could have used something else , they penalise us
    $endgroup$
    – user644728
    3 hours ago










  • $begingroup$
    "Can someone help me solve this limit?" And how is that supposed to help you learn?
    $endgroup$
    – John Coleman
    2 hours ago








1




1




$begingroup$
Looks like a case for the rule of L'Hospital. Have you tried that?
$endgroup$
– Cornman
3 hours ago




$begingroup$
Looks like a case for the rule of L'Hospital. Have you tried that?
$endgroup$
– Cornman
3 hours ago












$begingroup$
Hint: $sin x$ and $tan x$ can be replaced by $x$, to the first order. The limit is that of $(1+o(x))^{1/x}$.
$endgroup$
– Yves Daoust
3 hours ago






$begingroup$
Hint: $sin x$ and $tan x$ can be replaced by $x$, to the first order. The limit is that of $(1+o(x))^{1/x}$.
$endgroup$
– Yves Daoust
3 hours ago














$begingroup$
I have forgot to add the last part that if it is possible to not use L'Hospital , because if we use it in places where they think we could have used something else , they penalise us
$endgroup$
– user644728
3 hours ago




$begingroup$
I have forgot to add the last part that if it is possible to not use L'Hospital , because if we use it in places where they think we could have used something else , they penalise us
$endgroup$
– user644728
3 hours ago












$begingroup$
"Can someone help me solve this limit?" And how is that supposed to help you learn?
$endgroup$
– John Coleman
2 hours ago




$begingroup$
"Can someone help me solve this limit?" And how is that supposed to help you learn?
$endgroup$
– John Coleman
2 hours ago










3 Answers
3






active

oldest

votes


















1












$begingroup$

$hspace{5mm} lim_{xto 0}left(frac{1+tan(x)}{1+sin(x)}right)^{tfrac{1}{sin(x)}}$



$=lim_{xto 0}left(1+frac{1+tan(x)}{1+sin(x)}-1right)^{tfrac{1}{sin(x)}}$



$=lim_{xto 0}left(1+frac{tan(x)-sin(x)}{1+sin(x)}right)^{tfrac{1}{sin(x)}}$



$=lim_{xto 0}left(1+frac{1}{big(frac{1+sin(x)}{tan(x)-sin(x)}big)}right)^{tfrac{1}{sin(x)}}$



$=lim_{xto 0}left[left(1+frac{1}{big(frac{1+sin(x)}{tan(x)-sin(x)}big)}right)^{frac{1+sin(x)}{tan(x)-sin(x)}}right]^{tfrac{1}{sin(x)} cdot frac{tan(x)-sin(x)}{1+sin(x)}}$



$=mathrm{exp} big( {lim_{xto 0}tfrac{1}{sin(x)} frac{tan(x)-sin(x)}{1+sin(x)}} big)$



$=mathrm{exp} big( {lim_{xto 0}frac{1-cos(x)}{cos(x)(1+sin(x))}} big)$



$=mathrm{exp} big( {frac{1-1}{1(1+0)}} big)$



$=e^0$



$=1$






share|cite|improve this answer











$endgroup$





















    4












    $begingroup$

    The searched limit is the ratio



    $$frac{lim_{xto0}(1+tan x)^{1/sin x}}{lim_{xto0}(1+sin x)^{1/sin x}}=frac{lim_{xto0}((1+tan x)^{1/tan x})^{1/cos x}}{lim_{xto0}(1+sin x)^{1/sin x}}.$$



    As $tan x$ and $sin x$ tend to zero continuously with $x$, $$frac{e^1}e.$$






    share|cite|improve this answer











    $endgroup$





















      2












      $begingroup$

      Let
      $$L=lim_{xto 0} (frac{1+tan{(x)}}{1+sin{(x)}})^{frac{1}{sin{(x)}}}$$
      $$ln{L}=lim_{xto 0} frac{1}{sin{(x)}}ln{(frac{1+tan{(x)}}{1+sin{(x)}})}$$
      $$ln{L}=lim_{xto 0} frac{ln{(1+tan{(x)})}-ln{(1+sin{(x)})})}{sin{(x)}}$$
      $$ln{L}=lim_{xto 0} frac{frac{sec^2{(x)}}{1+tan{(x)}}-frac{cos{(x)}}{1+sin{(x)}}}{cos{(x)}}$$
      $$ln{L}=lim_{xto 0} frac{1}{cos^3{(x)}(1+tan{(x)})}-frac{1}{1+sin{(x)}}$$
      $$ln{L}= 1-1=0$$
      $$therefore L=e^0=1$$






      share|cite|improve this answer









      $endgroup$













        Your Answer





        StackExchange.ifUsing("editor", function () {
        return StackExchange.using("mathjaxEditing", function () {
        StackExchange.MarkdownEditor.creationCallbacks.add(function (editor, postfix) {
        StackExchange.mathjaxEditing.prepareWmdForMathJax(editor, postfix, [["$", "$"], ["\\(","\\)"]]);
        });
        });
        }, "mathjax-editing");

        StackExchange.ready(function() {
        var channelOptions = {
        tags: "".split(" "),
        id: "69"
        };
        initTagRenderer("".split(" "), "".split(" "), channelOptions);

        StackExchange.using("externalEditor", function() {
        // Have to fire editor after snippets, if snippets enabled
        if (StackExchange.settings.snippets.snippetsEnabled) {
        StackExchange.using("snippets", function() {
        createEditor();
        });
        }
        else {
        createEditor();
        }
        });

        function createEditor() {
        StackExchange.prepareEditor({
        heartbeatType: 'answer',
        autoActivateHeartbeat: false,
        convertImagesToLinks: true,
        noModals: true,
        showLowRepImageUploadWarning: true,
        reputationToPostImages: 10,
        bindNavPrevention: true,
        postfix: "",
        imageUploader: {
        brandingHtml: "Powered by u003ca class="icon-imgur-white" href="https://imgur.com/"u003eu003c/au003e",
        contentPolicyHtml: "User contributions licensed under u003ca href="https://creativecommons.org/licenses/by-sa/3.0/"u003ecc by-sa 3.0 with attribution requiredu003c/au003e u003ca href="https://stackoverflow.com/legal/content-policy"u003e(content policy)u003c/au003e",
        allowUrls: true
        },
        noCode: true, onDemand: true,
        discardSelector: ".discard-answer"
        ,immediatelyShowMarkdownHelp:true
        });


        }
        });






        user644728 is a new contributor. Be nice, and check out our Code of Conduct.










        draft saved

        draft discarded


















        StackExchange.ready(
        function () {
        StackExchange.openid.initPostLogin('.new-post-login', 'https%3a%2f%2fmath.stackexchange.com%2fquestions%2f3116681%2fcan-someone-help-me-solve-this-limit%23new-answer', 'question_page');
        }
        );

        Post as a guest















        Required, but never shown

























        3 Answers
        3






        active

        oldest

        votes








        3 Answers
        3






        active

        oldest

        votes









        active

        oldest

        votes






        active

        oldest

        votes









        1












        $begingroup$

        $hspace{5mm} lim_{xto 0}left(frac{1+tan(x)}{1+sin(x)}right)^{tfrac{1}{sin(x)}}$



        $=lim_{xto 0}left(1+frac{1+tan(x)}{1+sin(x)}-1right)^{tfrac{1}{sin(x)}}$



        $=lim_{xto 0}left(1+frac{tan(x)-sin(x)}{1+sin(x)}right)^{tfrac{1}{sin(x)}}$



        $=lim_{xto 0}left(1+frac{1}{big(frac{1+sin(x)}{tan(x)-sin(x)}big)}right)^{tfrac{1}{sin(x)}}$



        $=lim_{xto 0}left[left(1+frac{1}{big(frac{1+sin(x)}{tan(x)-sin(x)}big)}right)^{frac{1+sin(x)}{tan(x)-sin(x)}}right]^{tfrac{1}{sin(x)} cdot frac{tan(x)-sin(x)}{1+sin(x)}}$



        $=mathrm{exp} big( {lim_{xto 0}tfrac{1}{sin(x)} frac{tan(x)-sin(x)}{1+sin(x)}} big)$



        $=mathrm{exp} big( {lim_{xto 0}frac{1-cos(x)}{cos(x)(1+sin(x))}} big)$



        $=mathrm{exp} big( {frac{1-1}{1(1+0)}} big)$



        $=e^0$



        $=1$






        share|cite|improve this answer











        $endgroup$


















          1












          $begingroup$

          $hspace{5mm} lim_{xto 0}left(frac{1+tan(x)}{1+sin(x)}right)^{tfrac{1}{sin(x)}}$



          $=lim_{xto 0}left(1+frac{1+tan(x)}{1+sin(x)}-1right)^{tfrac{1}{sin(x)}}$



          $=lim_{xto 0}left(1+frac{tan(x)-sin(x)}{1+sin(x)}right)^{tfrac{1}{sin(x)}}$



          $=lim_{xto 0}left(1+frac{1}{big(frac{1+sin(x)}{tan(x)-sin(x)}big)}right)^{tfrac{1}{sin(x)}}$



          $=lim_{xto 0}left[left(1+frac{1}{big(frac{1+sin(x)}{tan(x)-sin(x)}big)}right)^{frac{1+sin(x)}{tan(x)-sin(x)}}right]^{tfrac{1}{sin(x)} cdot frac{tan(x)-sin(x)}{1+sin(x)}}$



          $=mathrm{exp} big( {lim_{xto 0}tfrac{1}{sin(x)} frac{tan(x)-sin(x)}{1+sin(x)}} big)$



          $=mathrm{exp} big( {lim_{xto 0}frac{1-cos(x)}{cos(x)(1+sin(x))}} big)$



          $=mathrm{exp} big( {frac{1-1}{1(1+0)}} big)$



          $=e^0$



          $=1$






          share|cite|improve this answer











          $endgroup$
















            1












            1








            1





            $begingroup$

            $hspace{5mm} lim_{xto 0}left(frac{1+tan(x)}{1+sin(x)}right)^{tfrac{1}{sin(x)}}$



            $=lim_{xto 0}left(1+frac{1+tan(x)}{1+sin(x)}-1right)^{tfrac{1}{sin(x)}}$



            $=lim_{xto 0}left(1+frac{tan(x)-sin(x)}{1+sin(x)}right)^{tfrac{1}{sin(x)}}$



            $=lim_{xto 0}left(1+frac{1}{big(frac{1+sin(x)}{tan(x)-sin(x)}big)}right)^{tfrac{1}{sin(x)}}$



            $=lim_{xto 0}left[left(1+frac{1}{big(frac{1+sin(x)}{tan(x)-sin(x)}big)}right)^{frac{1+sin(x)}{tan(x)-sin(x)}}right]^{tfrac{1}{sin(x)} cdot frac{tan(x)-sin(x)}{1+sin(x)}}$



            $=mathrm{exp} big( {lim_{xto 0}tfrac{1}{sin(x)} frac{tan(x)-sin(x)}{1+sin(x)}} big)$



            $=mathrm{exp} big( {lim_{xto 0}frac{1-cos(x)}{cos(x)(1+sin(x))}} big)$



            $=mathrm{exp} big( {frac{1-1}{1(1+0)}} big)$



            $=e^0$



            $=1$






            share|cite|improve this answer











            $endgroup$



            $hspace{5mm} lim_{xto 0}left(frac{1+tan(x)}{1+sin(x)}right)^{tfrac{1}{sin(x)}}$



            $=lim_{xto 0}left(1+frac{1+tan(x)}{1+sin(x)}-1right)^{tfrac{1}{sin(x)}}$



            $=lim_{xto 0}left(1+frac{tan(x)-sin(x)}{1+sin(x)}right)^{tfrac{1}{sin(x)}}$



            $=lim_{xto 0}left(1+frac{1}{big(frac{1+sin(x)}{tan(x)-sin(x)}big)}right)^{tfrac{1}{sin(x)}}$



            $=lim_{xto 0}left[left(1+frac{1}{big(frac{1+sin(x)}{tan(x)-sin(x)}big)}right)^{frac{1+sin(x)}{tan(x)-sin(x)}}right]^{tfrac{1}{sin(x)} cdot frac{tan(x)-sin(x)}{1+sin(x)}}$



            $=mathrm{exp} big( {lim_{xto 0}tfrac{1}{sin(x)} frac{tan(x)-sin(x)}{1+sin(x)}} big)$



            $=mathrm{exp} big( {lim_{xto 0}frac{1-cos(x)}{cos(x)(1+sin(x))}} big)$



            $=mathrm{exp} big( {frac{1-1}{1(1+0)}} big)$



            $=e^0$



            $=1$







            share|cite|improve this answer














            share|cite|improve this answer



            share|cite|improve this answer








            edited 3 hours ago









            Lemniscate

            402211




            402211










            answered 3 hours ago









            user326159user326159

            1,2521722




            1,2521722























                4












                $begingroup$

                The searched limit is the ratio



                $$frac{lim_{xto0}(1+tan x)^{1/sin x}}{lim_{xto0}(1+sin x)^{1/sin x}}=frac{lim_{xto0}((1+tan x)^{1/tan x})^{1/cos x}}{lim_{xto0}(1+sin x)^{1/sin x}}.$$



                As $tan x$ and $sin x$ tend to zero continuously with $x$, $$frac{e^1}e.$$






                share|cite|improve this answer











                $endgroup$


















                  4












                  $begingroup$

                  The searched limit is the ratio



                  $$frac{lim_{xto0}(1+tan x)^{1/sin x}}{lim_{xto0}(1+sin x)^{1/sin x}}=frac{lim_{xto0}((1+tan x)^{1/tan x})^{1/cos x}}{lim_{xto0}(1+sin x)^{1/sin x}}.$$



                  As $tan x$ and $sin x$ tend to zero continuously with $x$, $$frac{e^1}e.$$






                  share|cite|improve this answer











                  $endgroup$
















                    4












                    4








                    4





                    $begingroup$

                    The searched limit is the ratio



                    $$frac{lim_{xto0}(1+tan x)^{1/sin x}}{lim_{xto0}(1+sin x)^{1/sin x}}=frac{lim_{xto0}((1+tan x)^{1/tan x})^{1/cos x}}{lim_{xto0}(1+sin x)^{1/sin x}}.$$



                    As $tan x$ and $sin x$ tend to zero continuously with $x$, $$frac{e^1}e.$$






                    share|cite|improve this answer











                    $endgroup$



                    The searched limit is the ratio



                    $$frac{lim_{xto0}(1+tan x)^{1/sin x}}{lim_{xto0}(1+sin x)^{1/sin x}}=frac{lim_{xto0}((1+tan x)^{1/tan x})^{1/cos x}}{lim_{xto0}(1+sin x)^{1/sin x}}.$$



                    As $tan x$ and $sin x$ tend to zero continuously with $x$, $$frac{e^1}e.$$







                    share|cite|improve this answer














                    share|cite|improve this answer



                    share|cite|improve this answer








                    edited 2 hours ago

























                    answered 3 hours ago









                    Yves DaoustYves Daoust

                    128k674227




                    128k674227























                        2












                        $begingroup$

                        Let
                        $$L=lim_{xto 0} (frac{1+tan{(x)}}{1+sin{(x)}})^{frac{1}{sin{(x)}}}$$
                        $$ln{L}=lim_{xto 0} frac{1}{sin{(x)}}ln{(frac{1+tan{(x)}}{1+sin{(x)}})}$$
                        $$ln{L}=lim_{xto 0} frac{ln{(1+tan{(x)})}-ln{(1+sin{(x)})})}{sin{(x)}}$$
                        $$ln{L}=lim_{xto 0} frac{frac{sec^2{(x)}}{1+tan{(x)}}-frac{cos{(x)}}{1+sin{(x)}}}{cos{(x)}}$$
                        $$ln{L}=lim_{xto 0} frac{1}{cos^3{(x)}(1+tan{(x)})}-frac{1}{1+sin{(x)}}$$
                        $$ln{L}= 1-1=0$$
                        $$therefore L=e^0=1$$






                        share|cite|improve this answer









                        $endgroup$


















                          2












                          $begingroup$

                          Let
                          $$L=lim_{xto 0} (frac{1+tan{(x)}}{1+sin{(x)}})^{frac{1}{sin{(x)}}}$$
                          $$ln{L}=lim_{xto 0} frac{1}{sin{(x)}}ln{(frac{1+tan{(x)}}{1+sin{(x)}})}$$
                          $$ln{L}=lim_{xto 0} frac{ln{(1+tan{(x)})}-ln{(1+sin{(x)})})}{sin{(x)}}$$
                          $$ln{L}=lim_{xto 0} frac{frac{sec^2{(x)}}{1+tan{(x)}}-frac{cos{(x)}}{1+sin{(x)}}}{cos{(x)}}$$
                          $$ln{L}=lim_{xto 0} frac{1}{cos^3{(x)}(1+tan{(x)})}-frac{1}{1+sin{(x)}}$$
                          $$ln{L}= 1-1=0$$
                          $$therefore L=e^0=1$$






                          share|cite|improve this answer









                          $endgroup$
















                            2












                            2








                            2





                            $begingroup$

                            Let
                            $$L=lim_{xto 0} (frac{1+tan{(x)}}{1+sin{(x)}})^{frac{1}{sin{(x)}}}$$
                            $$ln{L}=lim_{xto 0} frac{1}{sin{(x)}}ln{(frac{1+tan{(x)}}{1+sin{(x)}})}$$
                            $$ln{L}=lim_{xto 0} frac{ln{(1+tan{(x)})}-ln{(1+sin{(x)})})}{sin{(x)}}$$
                            $$ln{L}=lim_{xto 0} frac{frac{sec^2{(x)}}{1+tan{(x)}}-frac{cos{(x)}}{1+sin{(x)}}}{cos{(x)}}$$
                            $$ln{L}=lim_{xto 0} frac{1}{cos^3{(x)}(1+tan{(x)})}-frac{1}{1+sin{(x)}}$$
                            $$ln{L}= 1-1=0$$
                            $$therefore L=e^0=1$$






                            share|cite|improve this answer









                            $endgroup$



                            Let
                            $$L=lim_{xto 0} (frac{1+tan{(x)}}{1+sin{(x)}})^{frac{1}{sin{(x)}}}$$
                            $$ln{L}=lim_{xto 0} frac{1}{sin{(x)}}ln{(frac{1+tan{(x)}}{1+sin{(x)}})}$$
                            $$ln{L}=lim_{xto 0} frac{ln{(1+tan{(x)})}-ln{(1+sin{(x)})})}{sin{(x)}}$$
                            $$ln{L}=lim_{xto 0} frac{frac{sec^2{(x)}}{1+tan{(x)}}-frac{cos{(x)}}{1+sin{(x)}}}{cos{(x)}}$$
                            $$ln{L}=lim_{xto 0} frac{1}{cos^3{(x)}(1+tan{(x)})}-frac{1}{1+sin{(x)}}$$
                            $$ln{L}= 1-1=0$$
                            $$therefore L=e^0=1$$







                            share|cite|improve this answer












                            share|cite|improve this answer



                            share|cite|improve this answer










                            answered 3 hours ago









                            Peter ForemanPeter Foreman

                            2,13613




                            2,13613






















                                user644728 is a new contributor. Be nice, and check out our Code of Conduct.










                                draft saved

                                draft discarded


















                                user644728 is a new contributor. Be nice, and check out our Code of Conduct.













                                user644728 is a new contributor. Be nice, and check out our Code of Conduct.












                                user644728 is a new contributor. Be nice, and check out our Code of Conduct.
















                                Thanks for contributing an answer to Mathematics Stack Exchange!


                                • Please be sure to answer the question. Provide details and share your research!

                                But avoid



                                • Asking for help, clarification, or responding to other answers.

                                • Making statements based on opinion; back them up with references or personal experience.


                                Use MathJax to format equations. MathJax reference.


                                To learn more, see our tips on writing great answers.




                                draft saved


                                draft discarded














                                StackExchange.ready(
                                function () {
                                StackExchange.openid.initPostLogin('.new-post-login', 'https%3a%2f%2fmath.stackexchange.com%2fquestions%2f3116681%2fcan-someone-help-me-solve-this-limit%23new-answer', 'question_page');
                                }
                                );

                                Post as a guest















                                Required, but never shown





















































                                Required, but never shown














                                Required, but never shown












                                Required, but never shown







                                Required, but never shown

































                                Required, but never shown














                                Required, but never shown












                                Required, but never shown







                                Required, but never shown







                                Popular posts from this blog

                                Hivernacle

                                Fluorita

                                Hulsita